Difference between revisions of "1994 AHSME Problems/Problem 13"
(→Solution) |
m (→Solution) |
||
Line 32: | Line 32: | ||
So <math>2x=\frac{180-x}{2}\implies 5x=180\implies x=\angle A=\boxed{\textbf{(B) }36^\circ.}</math> | So <math>2x=\frac{180-x}{2}\implies 5x=180\implies x=\angle A=\boxed{\textbf{(B) }36^\circ.}</math> | ||
+ | |||
--Solution by [http://www.artofproblemsolving.com/Forum/memberlist.php?mode=viewprofile&u=200685 TheMaskedMagician] | --Solution by [http://www.artofproblemsolving.com/Forum/memberlist.php?mode=viewprofile&u=200685 TheMaskedMagician] |
Revision as of 18:34, 20 July 2014
Problem
In triangle , . If there is a point strictly between and such that , then
Solution
Let . Since , we have as well. Then . Since , we have .
So
--Solution by TheMaskedMagician